Esercizio SISSA 2015

Messaggioda onlynose » 06/04/2019, 15:17

Ciao a tutti, sono nuovo nel forum. Vorrei proporvi un esercizio preso dal tema d'esame per l'ammissione della Laurea Magristale alla SISSA. Scusate per eventuali problemi di scrittura o quant'altro.

(a) Sia $f:(0,+\infty)\rightarrow\mathbb{R}$ una funzione differenziabile con derivata $f'$ uniformemente
continua su $(0,+\infty)$. Provare che se esiste il limite $\lim_{x\rightarrow+\infty} f(x) = L$ finito allora $\lim_{x\rightarrow+\infty} f'(x) = 0$.

(b) Dire se la conclusione precedente continua a valere assumendo solo che $f'$ è di classe
$C^1((0, +∞), R)$.

Il punto (b) è chiaramente falso, per vederlo considero la funzione $f(x)=\sin(x^2)/x$: si ha che $\lim_{x\rightarrow+\infty} f(x) =0$, mentre si ha $f'(x)=2\cos(x^2)-\sin(x^2)/x^2$, la quale è continua in $(0,+\infty)$ e si ha che non esiste il limite $\lim_{x\rightarrow+\infty} f'(x)$.

Mentre non sono riuscito a dimostrare il punto (a), ho pensato di ragionare per assurdo. Se il limite di $f'(x)$ esiste per $x\rightarrow\+infty$ si ottiene banalmente che tale limite è $0$.
Mentre se suppongo che il limite non esiste sono riuscito a provare che esiste una sottosuccessione convergente $x_n$ tale che $\lim_{n\rightarrow+\infty} f'(x_n) = 0$, usando lo stesso ragionamento usato nel caso in cui il limite esiste.

Vorrei qualche suggerimento su come usare l'ipotesi di uniforme continuità per provare il risultato (a).

Grazie a tutti!
onlynose
New Member
New Member
 
Messaggio: 1 di 72
Iscritto il: 02/04/2019, 19:12

Re: Esercizio SISSA 2015

Messaggioda Vincent46 » 07/04/2019, 10:24

Hintino:
Testo nascosto, fai click qui per vederlo
Pensa al controesempio del tuo punto $b$. Cosa comporta la mancanza dell'uniforme continuità? La derivata si allontana da $0$ infinite volte, (ad esempio, assume infinite volte il valore $1$). Però, scorrendo verso destra i punti dove vale $1$, succede che il "delta della continuità" diventa sempre più piccolo. Se chiedi che $f'$ sia uniformemente continua questo non può succedere.

Più esplicitamente:
Testo nascosto, fai click qui per vederlo
Se supponi che la tesi sia falsa, allora, per ogni $\varepsilon >0$, esiste una successione di punti ${x_n} \to \infty$ tale che $|f'(x_n)| \geq \varepsilon$ in tutto un intornino degli $x_n$ ampio $\delta$ (l'uniforme continuità ti serve per trovare un $\delta$ indipendente da $n$). Da qui giungi a un assurdo
Vincent46
Average Member
Average Member
 
Messaggio: 227 di 523
Iscritto il: 26/01/2014, 17:27

Re: Esercizio SISSA 2015

Messaggioda onlynose » 07/04/2019, 11:34

Si, ciò che hai detto avevo pensato, ovvero che esiste una sottosuccessione ${x_n}$ crescente tale che $f'(x_n)>\varepsilon$ (o viceversa $f'(x_n)<-\varepsilon$ ) in un intorno $(x_n-\delta,x_n+\delta)$ per ogni $n\ge1$ e per un certo $\varepsilon>0$ fissato.
Allora potrei provare a concludere così:

$$\sum_{n=1}^{+\infty}{\int_{x_n-\delta}^{x_n+\delta}f'(x)dx}>\sum_{n=1}^{+\infty}{\int_{x_n-\delta}^{x_n+\delta}\varepsilon dx} =\sum_{n=1}^{+\infty} 2\varepsilon\delta$$

che diverge.
Si ha che

$$\int_{x_1-\delta}^{+\infty}f'(x)dx\ge\sum_{n=1}^{+\infty}{\int_{x_n-\delta}^{x_n+\delta}f'(x)dx}$$

e l'integrale a primo membro vale per il teorema fondamentale del calcolo integrale $lim_{x\rightarrow+\infty}f(x)-f(x_1-\delta)=L-f(x_1-\delta)$, il quale è un numero finito e da qui nasce l'assurdo.

Secondo voi è giusto il mio ragionamento?

Riguardo al fatto che $\delta$ non dipende da $n$, ciò mi è garantito dall'uniforme continuità? Cioè: se così non fosse avrei due sottosuccesione arbitrariamente vicine tale che loro distanza sia maggiore di un certo $\varepsilon>0$, il che è assurdo, poiché significherebbe negare l'uniforme continuità. Dovrebbe funzionare così?
onlynose
New Member
New Member
 
Messaggio: 2 di 72
Iscritto il: 02/04/2019, 19:12

Re: Esercizio SISSA 2015

Messaggioda Vincent46 » 07/04/2019, 18:57

onlynose ha scritto:Si, ciò che hai detto avevo pensato, ovvero che esiste una sottosuccessione ${x_n}$ crescente tale che $f'(x_n)>\varepsilon$ (o viceversa $f'(x_n)<-\varepsilon$ ) in un intorno $(x_n-\delta,x_n+\delta)$ per ogni $n\ge1$ e per un certo $\varepsilon>0$ fissato.
Allora potrei provare a concludere così:

$$\sum_{n=1}^{+\infty}{\int_{x_n-\delta}^{x_n+\delta}f'(x)dx}>\sum_{n=1}^{+\infty}{\int_{x_n-\delta}^{x_n+\delta}\varepsilon dx} =\sum_{n=1}^{+\infty} 2\varepsilon\delta$$

che diverge.
Si ha che

$$\int_{x_1-\delta}^{+\infty}f'(x)dx\ge\sum_{n=1}^{+\infty}{\int_{x_n-\delta}^{x_n+\delta}f'(x)dx}$$

e l'integrale a primo membro vale per il teorema fondamentale del calcolo integrale $lim_{x\rightarrow+\infty}f(x)-f(x_1-\delta)=L-f(x_1-\delta)$, il quale è un numero finito e da qui nasce l'assurdo.

Secondo voi è giusto il mio ragionamento?

Riguardo al fatto che $\delta$ non dipende da $n$, ciò mi è garantito dall'uniforme continuità? Cioè: se così non fosse avrei due sottosuccesione arbitrariamente vicine tale che loro distanza sia maggiore di un certo $\varepsilon>0$, il che è assurdo, poiché significherebbe negare l'uniforme continuità. Dovrebbe funzionare così?


Questa formula:
$$\int_{x_1-\delta}^{+\infty}f'(x)dx\ge\sum_{n=1}^{+\infty}{\int_{x_n-\delta}^{x_n+\delta}f'(x)dx}$$
non mi torna... sarebbe senz'altro vera se $f'$ fosse sempre positiva, ma chi ti assicura che sia così?

Invece, degli intornini che hai trovato, prova a considerarne uno che sia abbastanza "in là" da assicurare che $|f(x) - L|$ sia abbastanza piccolo in tale intervallino. Quindi $f$ si deve mantenere abbastanza "piatta", cioé la derivata dovrebbe essere abbastanza vicina a zero. Tuttavia in quell'intervallino hai trovato che la derivata è maggiore di $\varepsilon$ (in valore assoluto. Quindi...

Testo nascosto, fai click qui per vederlo
Usa il fatto che, se l'intervallino è della forma $(x_0, x_1)$, si ha
$$ f(x_1) = f(x_0) + \int_{x_0}^{x_1} f'(x) dx.$$


Riguardo al fatto che δ non dipende da n, ciò mi è garantito dall'uniforme continuità?

È proprio la definizione di uniforme continuità applicata a ognuno dei punti $x_n$. La parte importante della definizione di continuità uniforme è esattamente che il $\delta$ non dipende dal punto.
Vincent46
Average Member
Average Member
 
Messaggio: 231 di 523
Iscritto il: 26/01/2014, 17:27

Re: Esercizio SISSA 2015

Messaggioda onlynose » 07/04/2019, 21:59

\[ 2\varepsilon>|f(x_2)-f(x_1)|=|\int_{x_1}^{x_2}f'(x)dx|\ge(x_2-x_1)\varepsilon_1=\delta\varepsilon_1 \]Grazio Vincent46 per i suggerimenti!
Provo a riformulare in un altro modo (sperando non sbagli nuovamente):

abbiamo detto che esiste $\varepsilon_1>0$ ed esiste una successione ${x_n}$ tale che $f'(x_n)\ge\varepsilon_1$ (oppure tale che $f'(x_n)\le\-varepsilon_1$) in un intorno di $x_n$ ampio $\delta$, che non dipende da $n$, per ogni $n\ge1$.

Per definizione di limite ho che qualsiasi $\varepsilon>0$ esiste $M>0$ tale che per ogni $x>M$ si abbia $|f(x)-L|<\varepsilon$. In particolare si avrà che $|f(x)-f(y)|<2\varepsilon$ per ogni $x,y>M$.

Allora sia $n^*\in\mathbb{N}$ tale che $x_{n^*}>M+\delta$. Da ciò considero un intorno di $x_{n^*}$ di raggio $\delta$, sia esso $(x_{n^*}-\delta/2,x_{n^*}+\delta/2)$. Pongo $x_1=x_{n^*}-\delta/2$ e $x_2=x_{n^*}+\delta/2$. Allora avrò che, poiché $x_1,x_2>M$

$$2\varepsilon>|f(x_2)-f(x_1)|=|\int_{x_1}^{x_2}f'(x)dx|\ge(x_2-x_1)\varepsilon_1=\delta\varepsilon_1$$.

Quindi infine avrei che $2\varepsilon\ge\delta\varepsilon_1$, il che è assurdo per l'arbitrarietà di $\varepsilon$.

Dici che così potrebbe andar bene?
onlynose
New Member
New Member
 
Messaggio: 4 di 72
Iscritto il: 02/04/2019, 19:12

Re: Esercizio SISSA 2015

Messaggioda Vincent46 » 07/04/2019, 23:48

:smt023
Vincent46
Average Member
Average Member
 
Messaggio: 235 di 523
Iscritto il: 26/01/2014, 17:27

Re: Esercizio SISSA 2015

Messaggioda dissonance » 08/04/2019, 07:59

Sicuramente va bene come hai fatto seguendo il suggerimento di Vincent, ma secondo me si può fare un po' prima osservando che
\[
f(x)=L-\int_x^\infty f'(y)\, dy.\]
Se il limite di \(f'\) esiste, diciamo \(c=\lim f'(x)\), e se \(c>0\), per \(x\) sufficientemente grande deve aversi \(f'(y)>c/2\), e quindi l'integrale è infinito, assurdo. Stesso discorso se \(c<0\). L'unica possibilità è che \(c=0\).
dissonance
Moderatore
Moderatore
 
Messaggio: 15219 di 27757
Iscritto il: 24/05/2008, 19:39
Località: Nomade

Re: Esercizio SISSA 2015

Messaggioda Vincent46 » 08/04/2019, 09:44

dissonance ha scritto:Sicuramente va bene come hai fatto seguendo il suggerimento di Vincent, ma secondo me si può fare un po' prima osservando che
\[
f(x)=L-\int_x^\infty f'(y)\, dy.\]
Se il limite di \(f'\) esiste, diciamo \(c=\lim f'(x)\), e se \(c>0\), per \(x\) sufficientemente grande deve aversi \(f'(y)>c/2\), e quindi l'integrale è infinito, assurdo. Stesso discorso se \(c<0\). L'unica possibilità è che \(c=0\).

Ma come si fa a dedurre che il limite esiste?
Vincent46
Average Member
Average Member
 
Messaggio: 236 di 523
Iscritto il: 26/01/2014, 17:27

Re: Esercizio SISSA 2015

Messaggioda dissonance » 08/04/2019, 11:17

Ah, mi sa che hai ragione, la traccia dice solo che esiste il limite di \(f\), \(f'\) a priori potrebbe non ammettere limite, avevo letto male.

Per completare questa dimostrazione tocca quindi dimostrare che \(f'\) ammette limite, il che presumibilmente ammonta a rifare l'argomento di Vincent46.
dissonance
Moderatore
Moderatore
 
Messaggio: 15221 di 27757
Iscritto il: 24/05/2008, 19:39
Località: Nomade

Re: Esercizio SISSA 2015

Messaggioda Bokonon » 08/04/2019, 21:37

Come al solito sono terra terra, specie a quest'ora. Ma se una funzione (così definita) converge a $+oo$ può solo ammettere un asintoto orizzontale. Se non ci fosse non convergerebbe (così come la derivata). E se fosse obliquio, non convergerebbe (però la derivata convergerebbe). Non vedo altre opzioni per il punto a).
Dove sbaglio?

P.S. Nella mia visione (fanciullesca) di asintoto includo anche le funzioni periodiche. Le "vedo" (in valore assoluto) come una pallina sottoposta a gravità che rimbalza per sempre su una superficie senza attrito :D
Avatar utente
Bokonon
Cannot live without
Cannot live without
 
Messaggio: 1051 di 5942
Iscritto il: 25/05/2018, 20:22

Prossimo

Torna a Pensare un po' di più

Chi c’è in linea

Visitano il forum: Nessuno e 1 ospite